Activist: Any member of the city council ought either to vote against the proposal or to abstain. But if all the memb...

C. Ann on March 25, 2015

Sufficient Assumption

Please explain this question. Thanks,

Replies
Create a free account to read and take part in forum discussions.

Already have an account? log in

Melody on March 27, 2015

This is a strengthen with sufficient premise question. Remember that a sufficient premise is sufficient for a conclusion, if and only if the existence of the premise guarantees or brings about the existence of the conclusion. Therefore, we need to find the premise that 100% guarantees the conclusion. The way you want to attack these answer choices is two-pronged. Ask yourself, does it strengthen? If it doesn't, then cross it out and continue to the next answer choice. If it does strengthen, however, then ask yourself whether or not the premise guarantees the conclusion.

Let's diagram:

"Any member of the city council ought either to vote against the proposal or to abstain."

P1: MCC ==> VAP or A
not VAP and not A ==> not MCC

"if all the members abstain, the matter will be decided by the city's voters"

P2: AMA ==> DBCV
not DBCV ==> not AMA

So at least one member of the city council should vote against the proposal.

C: not AMA

Answer choice (B) states: "The proposal should not be decided by the city's voters."

(B): not DBCV

Does this strengthen? Yes.

We know from the contrapositive of the second premise that if the matter is not decided by the city's voters, then not all the members have abstained. Answer choice (B) tells us that the proposal should not be decided by the city's voters. Thus, according to the contrapositive of "P2," not all members of the city council should abstain, i.e. the conclusion: at least one (which is the same as "not all") member of the city council should vote against the proposal.

Does the premise guarantee the conclusion? Yes.

This triggers the contrapositive of "P2," which leads us to "not AMA," i.e. not all members of the city council should vote against the proposal. Thus, answer choice (B) guarantees the conclusion of the argument.

Hope that was helpful! Please let us know if you have any other questions.

on September 10, 2019

I still don't see why D is wrong, can someone please explain?

Alexander on July 9, 2020

Can you please explain why A is wrong? The first sentence appears to indicate that it would be bad if the bill passed, and that is why the city shouldn't be allowed to vote for the motion - that is, they'd vote for it to pass.

Victoria on July 14, 2020

Hi @Minerva and @Alexander-Blankers,

Happy to help!

Answer choice (D) is incorrect because it is an illegal reversal of the second premise.

P2: AMA --> DBCV

(D): Not AMA --> Not DBCV

Answer choice (A) is definitely tempting. It does seem like the activist is suggesting that the proposal should not pass. However, the stimulus is not focused on whether the proposal should pass; rather, it is focused on whether the city's voters should be allowed to decide this.

While we could infer that the activist is suggesting that the proposal should not pass, it is less of a jump to infer that the activist is suggesting that the city's voters should not be allowed to make the decision.

Hope this helps! Please let us know if you have any further questions.